Is x > 0?

This topic has expert replies
Master | Next Rank: 500 Posts
Posts: 117
Joined: Wed Jun 09, 2010 7:02 am

Is x > 0?

by missrochelle » Sat Aug 14, 2010 7:33 am
Source: MGMAT

Is x > 0?

(1) |x + 3| < 4

(2) |x - 3| < 4

Answer: E

Can someone please explain the best way to look at the combined inequality? This is how MGMAT explains it:



If we combine the solutions from statements (1) and (2) we get an overlapping range of -1 < x < 1. We still can't tell whether x is positive.

IS THIS BECAUSE WE DON'T KNOW THAT X IS AN INTEGER, THEREFORE, COULD BE -1/4 OR 0 (WHICH IS NOT POS OR NEG)?

User avatar
Legendary Member
Posts: 1893
Joined: Sun May 30, 2010 11:48 pm
Thanked: 215 times
Followed by:7 members

by kvcpk » Sat Aug 14, 2010 7:43 am
missrochelle wrote: IS THIS BECAUSE WE DON'T KNOW THAT X IS AN INTEGER, THEREFORE, COULD BE -1/4 OR 0 (WHICH IS NOT POS OR NEG)?
you are right. its because x can still be 0 or negative or positive.
Since there are more than one options, the statement is INSUFFICIENT.

Hope this helps!!
"Once you start working on something,
don't be afraid of failure and don't abandon it.
People who work sincerely are the happiest."
Chanakya quotes (Indian politician, strategist and writer, 350 BC-275BC)

Legendary Member
Posts: 2326
Joined: Mon Jul 28, 2008 3:54 am
Thanked: 173 times
Followed by:2 members
GMAT Score:710

by gmatmachoman » Sat Aug 14, 2010 7:49 am
|x + 3| < 4

case 1: (x + 3 )< 4

= x <1

here X can be greater than zero or less than zero. SO Insufficient

case 2 : - ( x+3) <4
-x -3 <4
-x<7
x> -7

Here again X can be greater than Zero or less than zero.

So st 1 is insufficient.

st 2 : |x - 3| < 4

case 1 : x-3 <4
x< 7
We can have X greater than or less than 0 . Insufficient

case 2: -(x-3) <4
-x+3 <4
-x< 1
x>-1
Again we can have X greater than or lesser than 0.

Combining both the statements we have -1<x<7, the sts hold true. But again X can be greater than or less than zero
Example : X= -0.5

|x - 3| < 4

|-0.5-3| = 3.5 which is less than 4

Now X= 0.5
|0.5 -3|= 2.5 <4 YES.

Here we see , we get inconsistent answers.

so Pick E as -1<X<7